LSAT and Law School Admissions Forum

Get expert LSAT preparation and law school admissions advice from PowerScore Test Preparation.

 Administrator
PowerScore Staff
  • PowerScore Staff
  • Posts: 8917
  • Joined: Feb 02, 2011
|
#31836
Please post below with any questions!
 afulbright.2000@gmail.com
  • Posts: 13
  • Joined: Apr 28, 2021
|
#95512
Hi,

I just wanted to make sure there wasn't another way that I could have processed this answer more efficiently! I get all of the logic game questions correct when I give myself a little more time per game, but clearly, to achieve a stellar performance on the exam, speed and accuracy must go hand-in-hand.

For this problem, I took the time to write out the hypothetical:

1- Gw
Fo
2- Ho
**Fi**
3- Iw
Go
4- Hw
Io

The question for the problem was if Ho is displayed on wall 2, what could also be displayed on wall 2?

None of the answer choices had Fi as an option, so I looked at what other variable could be switched with Fi that would still fulfill the rules. I realized I could swap Fi with Iw to create this second hypothetical and keep G on top of F to still complete one of the initial rules.

1- Gw
Fo
2- Ho
** Iw**
3- Go
Fi
4- Hw
Io

Was the way I went about solving this question the most efficient way possible or could I have done this another way??
 Rachael Wilkenfeld
PowerScore Staff
  • PowerScore Staff
  • Posts: 1358
  • Joined: Dec 15, 2011
|
#95523
I'll be honest here, afulbright. I'm not sure where you are getting your hypotheticals. Are you just using some potential order that might work? I recommend against that---you want to be more targeted in your process. The risk with working from these sorts of open hypotheticals is that you risk assuming something MUST be true that only COULD be true.

For this one, I started with my main diagram, which has Io in the lower position of day 4, and Hw in the upper position of day 4. That's on our starting diagram.

I then added Ho under day 2. Who has to go with Ho? We know it has to be a watercolor based on rule 1 which eliminates answer choice (A) and answer choice (C) right away. We know it can't be from H based on rule 2 which eliminates answer choice (D). We know it can't be answer choice (B) based on rule 4---Gw is already used with another painting. That leaves answer choice (E) as the only possible answer.

From my initial set-up, I was able to quickly see that I was looking for a watercolor that was not G or H's. The only answer choice that fit was answer choice (E). Ideally, this question should be fairly quick and efficient to answer.

Hope that helps!
 afulbright.2000@gmail.com
  • Posts: 13
  • Joined: Apr 28, 2021
|
#95529
Very helpful, that was such a better way to approach this problem.

I've noticed that I have tendencies to make things more complicated than they need to be. I see how efficiently this problem should be solved by simply utilizing the rules from the beginning and will try to reinforce having that approach for future problems like this one. No need for me to spend the time conjuring up additional hypotheticals. :-D

Thank you so much!

Get the most out of your LSAT Prep Plus subscription.

Analyze and track your performance with our Testing and Analytics Package.